Korva population

This topic has expert replies
Legendary Member
Posts: 2326
Joined: Mon Jul 28, 2008 3:54 am
Thanked: 173 times
Followed by:2 members
GMAT Score:710

Korva population

by gmatmachoman » Sun Dec 06, 2009 3:52 am
Ditrama is a federation made up of three autonomous regions. Korva. Mitro, and Guadar, Under the federal revenue-sharing plan, each region receives a share of federal revenues equal to the share of the total population of Ditrama residing in that region as shown by a yearly population survey. Last year the percentage of federal revenues Korva received for its share decreased somewhat even though the population survey on which the revenue-sharing was based showed that Korva's population had increased.

If the statements above are true, which one of the following must also have been shown by the population survey on which last year's revenue-sharing in Dirama was based?

(A) Of the three regions Korva had the smallest number of residents
(B) The population of Korva grew by a smaller percentage than it did in previous years
(C) The populations of Mitro and Guadar each increased by a percentage that exceeded the percentage by which the population of Korva increased.
(D) Of the three regions. Korva's numerical increase in population was the smallest
(E) Korva's population grew by a smaller percentage than did the population of at least one of the other two autonomous regions.

OA: after discussion

Junior | Next Rank: 30 Posts
Posts: 18
Joined: Sat Nov 28, 2009 11:21 am
Location: London

by godspeed » Sun Dec 06, 2009 4:24 am
IMO its D

User avatar
Master | Next Rank: 500 Posts
Posts: 434
Joined: Mon Jun 11, 2007 9:48 pm
Location: Bangalore
Thanked: 6 times
GMAT Score:600

by viju9162 » Sun Dec 06, 2009 4:58 am
Is it A
"Native of" is used for a individual while "Native to" is used for a large group

Senior | Next Rank: 100 Posts
Posts: 74
Joined: Fri Oct 09, 2009 4:20 pm
Thanked: 4 times

by nakul_anand » Sun Dec 06, 2009 5:19 am
IMO its A.

The argument says that the share of the federal revenue for a particular region is directly propotional to the SHARE of the total population of Ditrama residing in that particular region. 'SHARE' is relative to the whole population of Ditrama.

So even if the population of Korva increased, if Korva had the lowest population among the three regions, then the share of federal revenue would be the lowest for Korva, irrespective of the popluation increase it had.

What's the OA?

Master | Next Rank: 500 Posts
Posts: 256
Joined: Mon Aug 10, 2009 6:31 pm
Thanked: 3 times

by gmatv09 » Sun Dec 06, 2009 8:48 am
IMO A...

GMAT Instructor
Posts: 1302
Joined: Mon Oct 19, 2009 2:13 pm
Location: Toronto
Thanked: 539 times
Followed by:164 members
GMAT Score:800

by Testluv » Sun Dec 06, 2009 7:08 pm
The correct answer must be choice E.

We know that "each region receives a share of federal revenues equal to the share of the total population " of the Ditrama federation. We also know that Korvo's population increased in number (pop survey discussed in last sentence) and that, surprisingly, its share went down.

The only way to reconcile these facts is that, collectively, the sum population of the other two regions increased by a greater number than did the population of Korvo. This is our key deduction/prediction in this inference question.

Choice E matches this. If you were debating between choices A and E, recognize that choice A is extreme ("smallest") while choice E is more tentative ("...at least one of..."). In an inference question, if you are struggling between two choices, and one is clealry more tentative (less extreme) than another, always go with the more tentative (less extreme) one.
Kaplan Teacher in Toronto

Senior | Next Rank: 100 Posts
Posts: 44
Joined: Tue Feb 03, 2009 5:17 am

by mridula » Wed Dec 09, 2009 12:34 pm
Testluv wrote:The correct answer must be choice E.

We know that "each region receives a share of federal revenues equal to the share of the total population " of the Ditrama federation. We also know that Korvo's population increased in number (pop survey discussed in last sentence) and that, surprisingly, its share went down.

The only way to reconcile these facts is that, collectively, the sum population of the other two regions increased by a greater number than did the population of Korvo. This is our key deduction/prediction in this inference question.

Choice E matches this. If you were debating between choices A and E, recognize that choice A is extreme ("smallest") while choice E is more tentative ("...at least one of..."). In an inference question, if you are struggling between two choices, and one is clealry more tentative (less extreme) than another, always go with the more tentative (less extreme) one.
Why is C wrong?

GMAT Instructor
Posts: 1302
Joined: Mon Oct 19, 2009 2:13 pm
Location: Toronto
Thanked: 539 times
Followed by:164 members
GMAT Score:800

by Testluv » Wed Dec 09, 2009 12:49 pm
mridula wrote:
Testluv wrote:The correct answer must be choice E.

We know that "each region receives a share of federal revenues equal to the share of the total population " of the Ditrama federation. We also know that Korvo's population increased in number (pop survey discussed in last sentence) and that, surprisingly, its share went down.

The only way to reconcile these facts is that, collectively, the sum population of the other two regions increased by a greater number than did the population of Korvo. This is our key deduction/prediction in this inference question.

Choice E matches this. If you were debating between choices A and E, recognize that choice A is extreme ("smallest") while choice E is more tentative ("...at least one of..."). In an inference question, if you are struggling between two choices, and one is clealry more tentative (less extreme) than another, always go with the more tentative (less extreme) one.
Why is C wrong?
C is wrong because it doesn't have to be true that BOTH of the other countries had a bigger increase. Instead, collectively their increase must have been bigger; in other words, at least one of them had to have had a greater increase. So, it could be the case that only one of them increased.
Kaplan Teacher in Toronto

Junior | Next Rank: 30 Posts
Posts: 11
Joined: Mon May 07, 2012 9:34 am
Location: Uzbekistan

by bzuhurov » Mon Sep 03, 2012 5:09 pm
Thanks a lot!!!!!!!!!!!!!!!!